Does the Taylor series for arctan converge at x = 1?

In summary, a series representation for π is a mathematical expression that represents the value of π as an infinite sum of terms. It can be derived using various mathematical techniques and is significant in approximating the value of π and understanding its relationship with other mathematical functions. There are other series representations for π, but their accuracy depends on the number of terms used. However, it is impossible to achieve a completely accurate representation due to the infinite nature of the series.
  • #1
Euge
Gold Member
MHB
POTW Director
2,052
207
Show that $$\frac{\pi}{4} = 1 - \frac{1}{3} + \frac{1}{5} - \frac{1}{7} + \cdots$$
 
  • Like
Likes Greg Bernhardt and topsquark
Physics news on Phys.org
  • #2
We have

\begin{align*}
\arctan (z) = z - \frac{z^3}{3} + \frac{z^5}{5} - \frac{z^7}{7} + \cdots = \sum_{n=0}^\infty \dfrac{(-1)^n z^{2n+1}}{2n+1} ; \quad |z| \leq 1 \quad z \not= \pm i
\end{align*}

So that

\begin{align*}
\frac{\pi}{4} = \arctan (1) = 1 - \frac{1}{3} + \frac{1}{5} - \frac{1}{7} + \cdots = \sum_{n=0}^\infty \dfrac{(-1)^n}{2n+1}
\end{align*}
 
  • Like
Likes ohwilleke and topsquark
  • #3
The expansion ##\arctan z=z-\frac{z^3}{3}+\ldots## follows from general theory for ##|z|<1.## The assertion ##\arctan(1)=1-\frac{1}{3}+\ldots## needs some additional work. Namely, one must show that the series in the right side converges to ##\arctan(1)## indeed.
 
  • Like
Likes dextercioby, julian and topsquark
  • #4
wrobel said:
The expansion ##\arctan z=z-\frac{z^3}{3}+\ldots## follows from general theory for ##|z|<1.## The assertion ##\arctan(1)=1-\frac{1}{3}+\ldots## needs some additional work. Namely, one must show that the series in the right side converges to ##\arctan(1)## indeed.
Because ##\dfrac{1}{1+z^2}## only converges for ##|z| < 1##?
 
  • #5
julian said:
Because ##\dfrac{1}{1+z^2}## only converges for ##|z| < 1##?
Because it is insufficient to prove that 1-1/3+... converges, one must prove that it converges to ##\arctan 1##
 
  • #6
wrobel said:
Because it is insufficient to prove that 1-1/3+... converges, one must prove that it converges to ##\arctan 1##
Well, since arctanz is Complex-analytic; entire actually, that should be enough.
 
Last edited:
  • #8
WWGD said:
Well, since arctanz is Complex-analytic; entire actually, that should be enough.
By which theorem is it enough? Consider me as a wet blanket but I think that if we are solving a math problem, we must refer to mathematical theorems and apply them. For example, the result follows from
Theorem (Abel). Assume we have an analytic function
$$f(z)=\sum_{k=0}^\infty a_kz^k,\quad a_k\in\mathbb{R}$$ and
the Taylor series is convergent in ##\{|z|<1\}##. If a series
##a=\sum_{k=0}^\infty a_k## converges then
$$\lim_{\mathbb{R}\ni x\to 1-}f(x)=a.$$
 
  • Like
Likes dextercioby and PeroK
  • #10
wrobel said:
By which theorem is it enough? Consider me as a wet blanket but I think that if we are solving a math problem, we must refer to mathematical theorems and apply them. For example, the result follows from
Theorem (Abel). Assume we have an analytic function
$$f(z)=\sum_{k=0}^\infty a_kz^k,\quad a_k\in\mathbb{R}$$ and
the Taylor series is convergent in ##\{|z|<1\}##. If a series
##a=\sum_{k=0}^\infty a_k## converges then
$$\lim_{\mathbb{R}\ni x\to 1-}f(x)=a.$$
I thought we were working over the Complexes, where analiticity is guaranteed by differentiability.
 
  • #11
WWGD said:
I thought we were working over the Complexes, where analiticity is guaranteed by differentiability.
We are working over complexes. Please present your argument as a formal proof.
 
  • Like
Likes PeroK
  • #12
wrobel said:
We are working over complexes. Please present your argument as a formal proof.
You mean that a differentiable function is infinitely-differentiable and analytic?
 
  • #13
Isn't this one of the mainstream results in Complex Analysis? Differentiability implies ##C^{\infty}##, implies Analytic?
 
  • #14
WWGD said:
You mean that a differentiable function is infinitely-differentiable and analytic?
I mean posts #5, 6
How does your answer from #6 solve the problem formulated in #5?
 
Last edited:
  • Like
Likes PeroK
  • #15
WWGD said:
Isn't this one of the mainstream results in Complex Analysis? Differentiability implies ##C^{\infty}##, implies Analytic?
The issue is whether the Taylor series for ##\arctan## converges to the function at the radius of convergence (that is to say at ##x = 1##). We can take that the Taylor series converges to the function value within the radius of convergence, but not at the radius itself. That's a standard result. In this case we have:
$$\arctan(x) = \sum_{n = 0}^{\infty} (-1)^n \frac{x^{2n+1}}{2n+1} \ \ \ (|x| < 1)$$The question is what happens at ##x = 1##? The series for ##x = 1## converges, by the alternating series test, but does it converge to ##\arctan(1)##?

Note that I posted a proof from Wiki above (post #9), that uses integration over the interval ##(0, 1)## to prove the equality at the endpoint.

I'd also be interested to see an alternative proof using complex numbers, if you could supply one.
 
  • Like
Likes WWGD
  • #16
We will evaluate

\begin{align*}
\sum_{k=1}^\infty \frac{(-1)^k \sin \dfrac{\pi k}{2}}{k} = - \sum_{k=1}^\infty \dfrac{(-1)^{k-1}}{2k-1}
\end{align*}

I will express the sum via a complex contour integration. It employs the fact that the function

\begin{align*}
\dfrac{\sin \dfrac{\pi z}{2}}{\sin \pi z}
\end{align*}

has simple poles at all integer values except when ##\sin \dfrac{\pi z}{2} = 0##.

This allows us to write

\begin{align*}
\sum_{k=1}^\infty \frac{(-1)^k \sin \dfrac{\pi k}{2}}{k} = \frac{1}{2i} \oint_C \dfrac{\sin \dfrac{\pi z}{2}}{z \sin \pi z} dz
\end{align*}

where the contour ##C## is defined in fig (a).
contoursum.jpg

We have

\begin{align*}
2 \sum_{k=1}^\infty (-1)^k \frac{\sin \dfrac{\pi k}{2}}{k} & = \sum_{k=1}^\infty (-1)^k \frac{\sin \dfrac{\pi k}{2}}{k} + \sum_{k=-1}^{-\infty} (-1)^k\frac{\sin \dfrac{\pi k}{2}}{k}
\nonumber \\
& = \frac{1}{2i} \oint_{C+C'} \dfrac{\sin \dfrac{\pi z}{2}}{z \sin \pi z} dz
\end{align*}

where the contour ##C'## is defined in fig (a). We complete the path of integration along semicircles at infinity (see fig (b)) since the integrand vanishes there. Since the resulting enclosed area contains no singularities except at ##z=0##, we can shrink this contour down to an infinitesimal circle ##C_0## around the origin (see fig (c)). So that

\begin{align*}
\sum_{k=1}^\infty (-1)^k \frac{\sin \dfrac{\pi k}{2}}{k} = \frac{1}{4 i} \oint_{C_0} \dfrac{\sin \dfrac{\pi z}{2}}{z \sin \pi z} dz
\end{align*}

We expand the integrand in powers of ##z## about ##z=0## and isolate the ##z^{-1}## term. We get

\begin{align*}
\dfrac{\sin \dfrac{\pi z}{2}}{z \sin \pi z} & = \dfrac{\frac{\pi z}{2} - \cdots}{z [\pi z - \frac{1}{3!} \pi^3 z^3 + \cdots]}
\nonumber \\
& = \dfrac{\frac{\pi z}{2}- \cdots}{z^2 \pi [1 - \frac{1}{6} \pi^2 z^2 + \cdots]}
\nonumber \\
& = \dfrac{\frac{\pi z}{2} - \cdots}{z^2 \pi} (1 + \frac{1}{6} \pi^2 z^2 + \cdots)
\nonumber \\
& = \frac{1}{2z} + \cdots
\end{align*}

So that,

\begin{align*}
\sum_{k=1}^\infty (-1)^k \frac{\sin \dfrac{\pi k}{2}}{k} & = \frac{1}{4 i} \oint_{C_0} \dfrac{\sin \dfrac{\pi z}{2}}{z \sin \pi z} dz
\nonumber \\
& = \frac{1}{4 i} (-2 \pi i) \frac{1}{2}
\nonumber \\
& = - \frac{\pi}{4} .
\end{align*}

So that

\begin{align*}
\frac{\pi}{4} = \sum_{k=1}^\infty \dfrac{(-1)^{k-1}}{2k-1} .
\end{align*}
 
  • Wow
Likes PeroK
  • #17
I think that the shortest proof is via the Taylor series for arctan and Abel's theorem.
 
  • #18
Given a circle and a square which circumscribes that circle, the ratio of areas of that circle to that square is ##\frac{\pi}{4}##.
I wonder if there is a purely geometric construction based series approach to this problem?
 
  • Like
Likes wrobel
  • #19
PeroK said:
The issue is whether the Taylor series for ##\arctan## converges to the function at the radius of convergence (that is to say at ##x = 1##). We can take that the Taylor series converges to the function value within the radius of convergence, but not at the radius itself. That's a standard result. In this case we have:
$$\arctan(x) = \sum_{n = 0}^{\infty} (-1)^n \frac{x^{2n+1}}{2n+1} \ \ \ (|x| < 1)$$The question is what happens at ##x = 1##? The series for ##x = 1## converges, by the alternating series test, but does it converge to ##\arctan(1)##?

Note that I posted a proof from Wiki above (post #9), that uses integration over the interval ##(0, 1)## to prove the equality at the endpoint.

I'd also be interested to see an alternative proof using complex numbers, if you could supply one.
Thanks for clarifying. I missed that obvious point( not being sarcastic) . Embarrassed.
 

1. What is the Taylor series for arctan?

The Taylor series for arctan is an infinite series representation of the arctangent function, which is defined as the inverse of the tangent function. It is given by the formula: arctan(x) = x - (x^3)/3 + (x^5)/5 - (x^7)/7 + ...

2. How is the Taylor series for arctan derived?

The Taylor series for arctan is derived by using the Taylor series expansion formula, which is based on the derivatives of a function evaluated at a specific point. In this case, the derivatives of the arctangent function are evaluated at x = 0, and the resulting series is centered at x = 0.

3. Does the Taylor series for arctan converge for all values of x?

No, the Taylor series for arctan only converges for values of x within the interval [-1, 1]. This is because the arctangent function is undefined for values outside of this interval.

4. Does the Taylor series for arctan converge at x = 1?

Yes, the Taylor series for arctan does converge at x = 1. This can be shown by substituting x = 1 into the series and using the alternating series test, which states that if the terms of an alternating series decrease in absolute value and approach 0, then the series converges.

5. How accurate is the Taylor series for arctan at x = 1?

The accuracy of the Taylor series for arctan at x = 1 depends on the number of terms used in the series. The more terms that are included, the closer the approximation will be to the actual value of arctan(1), which is equal to π/4. However, even with a large number of terms, the series will never be exact due to the nature of infinite series.

Similar threads

  • Math POTW for University Students
Replies
1
Views
463
  • Math POTW for University Students
Replies
3
Views
790
  • Math POTW for University Students
Replies
1
Views
548
  • Calculus and Beyond Homework Help
Replies
1
Views
261
  • Calculus and Beyond Homework Help
Replies
3
Views
419
  • Calculus and Beyond Homework Help
Replies
1
Views
221
  • Calculus and Beyond Homework Help
Replies
2
Views
519
  • Math POTW for University Students
Replies
1
Views
2K
  • Calculus and Beyond Homework Help
Replies
2
Views
190
  • Calculus and Beyond Homework Help
Replies
2
Views
739
Back
Top